SlideShare a Scribd company logo
1 of 15
Download to read offline
Section III – Musculoskeletal Radiology




                                                  Figure 1A




                                                  Figure 1B

90.   A 15-year-old woman presents with aching knee pain and no history of trauma. You are shown an AP radiograph
      (Figure 1A) and non-contrast CT image (Figure 1B). Which one of the following is the MOST likely diagnosis?
      A. Osteomyelitis
      B.   Osteosarcoma
      C. Chondroblastoma
      D. Osteoid Osteoma
      E.   Ewing’s Sarcoma

                                  Diagnostic In-Training Exam 2003                                                  1
Section III – Musculoskeletal Radiology
Question #90
Findings: The standing AP radiograph of the femur shows a very dense but smooth and benign appearing periosteal
reaction at the medial aspect of the distal femoral Meta diaphysis. No destructive or aggressive features are present on
the image. The single axial, non-contrast CT image shows the dense reactive bone involving the medial and posterior
cortex as well as a small lucent nidus with central calcification within the cortex itself. There is no soft tissue mass or
any disruption of soft tissue planes on the CT image.

Rationales:
A) Incorrect. The pattern of osteomyelitis depends on the age of the patient and the mode of infection. In infants,
   perforating vessels cross the open growth plate and hematogenously spread infection can extend to the epiphysis.
   In childhood and early adolescence, those perforating vessels regress and there is not communication across the
   open growth plate and infection more commonly involves the metaphysis or Meta diaphysis. As the growth plate
   closes, there are again patent vessels, which allow communication between the metaphysis and epiphysis, but it
   is far more common for adults to acquire osteomyelitis from a source of infection in the contiguous soft tissues
   rather than from a hematogenous source. Radiographically, osteomyelitis usually presents as lucency in the bone.
   Depending on the infectious agent and the chronicity of infection, there may be more or less reactive bone near
   the lesion. With subacute or chronic infection, well-defined intraosseous abscesses with a sclerotic margin may
   develop (Brodie’s abscess) and sinus tracts may be seen. Bony sequestra, which are intracortical pieces of necrotic
   bone surrounded by granulation tissue, may also be seen. In the case of bone infection from an adjacent soft
   tissue source, the key diagnostic feature distinguishing osteomyelitis from reactive change to cellulitis is cortical
   destruction. In the early phases of infection, subtle changes in the bone may be imperceptible radiographically.
   MRI is both more sensitive and more specific for the bone changes in addition to identifying the accompanying
   soft tissue abnormalities. The specificity of MRI may be improved by the addition of nuclear medicine studies
   using labeled white cells to localize infection. In our case, the calcified nidus might possibly be taken for a small
   sequestrum and osteomyelitis is a differential diagnostic consideration here. However, the growth plate in this
   adolescent girl is closed, making the metaphysis an unlikely location for infection in this patient and the dense,
   benign reactive bone would be atypical for even the most chronic infection.
B) Incorrect. Osteosarcoma is second only to myeloma in frequency as a primary malignancy of bone. Commonly
   presenting in the second and third decades, approximately 50-75% of the most common variant, conventional
   osteosarcoma, occurs about the knee. The tumor typically has an aggressive appearance, with destruction of the
   underlying bone and variable production of malignant appearing osteoid. Radiographically osteosarcoma is an
   ill-defined, destructive intramedullary, metaphyseal lesion with an associated soft tissue mass. Due to rapid
   growth, there is commonly “sunburst” periostitis or Codman’s triangles. X –rays are preferred for the initial
   diagnosis, but MR is superior to CT in the evaluation of the intra and extraosseous extent of disease. The
   metaphyseal location and age of the patient in question would be appropriate for osteosarcoma, but the smooth,
   benign reactive bone and lack of any destructive changes or a soft tissue mass are consistent with a non-
   aggressive process.
C) Incorrect. Chondroblastoma is a rare, benign primary tumor of bone that most commonly seen in the second
   decade. It typically presents as a lytic lesion in the epiphysis or apophysis of a long bone with a well-defined
   sclerotic margin. Matrix calcification is present in up to 50%. MRI may show edema in the surrounding bone
   and soft tissues due to the prostaglandins secreted by the tumor and clinically they may mimic osteoid osteoma.
   Benign periostitis or joint effusions may be seen, most commonly when the lesion is located in the capital
   femoral epiphysis within the hip joint capsule. The lesion may appear expansile, most commonly when a
   secondary aneurysmal bone cyst is coexistent. Rarely chondroblastoma may metastasize to the lungs. Treatment
   is usually curettage with bone grafting with image-guided radiofrequency ablation being used at some centers.
   Although the age of the patient in the index case would be appropriate for chondroblastoma, the location in the
   Meta diaphysis of the femur and the large amount of reactive bone would not be.




2                                        American College of Radiology
Section III – Musculoskeletal Radiology
D) Correct. Osteoid osteoma is a benign bone-forming neoplasm consisting of a central core of vascularized
   osteoid surrounded by densely sclerotic bone. The clinical history is often suggestive, with pain, which is worse
   at night and relieved by prostaglandin inhibiting agents such as aspirin. Age at presentation is usually in the
   second or third decade. Lesions in long bones are commonly cortically based where they typically present as
   lucency, the “nidus”, which may or may not contain calcification. The nidus is usually located at the center of
   the reactive sclerotic bone. In the small bones of the hands and feet, the lesions tend to be intramedullary with
   an associated periosteal reaction. Subperiosteal lesions can be seen and may have less prominent reactive changes.
   The most common location is in the long tubular bones, typically in the diaphysis or metaphysis. Vertebral
   lesions, often associated with a painful scoliosis, are usually located in the posterior elements. MR imaging of
   osteoid osteoma shows edema in the bone and soft tissues, which may be deceptively aggressive in appearance.
   Scintigraphy has been used in the past for its high sensitivity but it remains low in specificity. Plain films usually
   show the benign reactive bone. High resolution CT is best for showing the nidus itself. CT may also be used
   for pre-operative localization or definitive treatment with radiofrequency ablation. Our case shows the classic
   appearance of osteoid osteoma with the dense but benign reactive bone with a subtle lucency on x-ray. The CT
   demonstrates a well-defined nidus with central calcification. This case was subsequently successfully treated with
   CT-guided radiofrequency ablation with complete resolution of symptoms.
E) Incorrect. Ewing’s Sarcoma is a primary malignancy of bone that chiefly affects young children, often under the
   age of 10. It most commonly affects the femur and in general is more common in the lower part of the body.
   In long bones, the Meta diaphysis or diaphysis are typical locations in the bone. Radiographically the lesion is
   primarily lytic and may have a permeative appearance that may be mistaken for infection. A malignant periosteal
   reaction is present which may appear laminated (“onion-skin” pattern) and a large soft tissue mass is usually
   seen. The lesion appears central in the bone reflecting its origin from bone marrow. Ewing’s Sarcoma often
   presents with constitutional symptoms such as fever, which may delay diagnosis. Scintigraphy is sensitive but
   non-specific. Radiographs are the usual modality for primary diagnosis with MRI showing the extent of disease
   within the bone marrow and any associated soft tissue mass. MRI is also often used to monitor response to
   treatment with chemotherapy. CT is especially helpful when flat bones such as the pelvis or skull are involved.
   In our case, the lesion is cortically based with a very benign and dense periosteal reaction. This would not be
   consistent with a malignant process. The CT scan shows no soft tissue mass or disruption of soft tissue planes.

Citations:
Torriani M, Rosenthal DL. Percutaneous radiofrequency ablation of osteoid osteoma. Pediatric Radiology 32(8):
    615-8, 2002.
Ho AC, Horton KM, McCarthy EF, Fishman EK. The role of imaging in the diagnosis and management of osteoid
   osteoma: a pictorial review. Critical Reviews in Diagnostic Imaging 42(6): 357-77, 2001.




                                    Diagnostic In-Training Exam 2003                                                  3
Section III – Musculoskeletal Radiology




                                                   Figure 2


91.   You are shown an axial T2-weighted, fat suppressed MR image in a 25-year-old woman who presents with
      pain after kicking the ball while playing soccer (Figure 2). Which one of the following is the MOST likely
      diagnosis?
      A. Hamstring avulsion
      B.   Insufficiency fracture
      C. Adductor strain
      D. Gluteal myositis
      E.   Denervation injury




4                                      American College of Radiology
Section III – Musculoskeletal Radiology
Question #91
Findings: The image presented is a fat suppressed, T2 weighted axial MR image of the lower pelvis at the level of
the ischial tuberosities. The image shows a focal area of increased T2 signal between the hamstring tendons on the
right and the tip of the ischial tuberosity. The marrow signal in both inferior pubic rami is normal, as is the signal
in the musculature.

Rationales:
A) Correct. Hamstring injuries are commonly seen in athletes such as hurdlers or those who participate in sports
   with powerful kicking. In children, these injuries are often associated with avulsions of the ischial apophysis; in
   young adults, tendon avulsions without underlying fractures may be seen. Radiographs are often normal in the
   absence of an associated fracture. MRI will show the relationship of the tendons to their attachment, associated
   muscle injuries, and the presence or absence of osseous pathology. In our case, the T2 weighted fat suppressed
   axial image shows high signal fluid between the hamstring tendons and the bone, with no such separation on
   the contralateral normal side. The inferior pubic rami are normal on both sides with no evidence for fracture.
   The muscles themselves are also normal. There is some fluid surrounding the right sciatic nerve, which is
   otherwise normal, explaining why this patient may present with complaints of sciatica due to irritation of the
   nerve related to its proximity to the tendon avulsion. These injuries are most often treated conservatively with
   the exception of apophyseal avulsions in childhood, which may require fixation.
B) Incorrect. Insufficiency fractures are the result of normal stresses on bone that has lost its normal elastic
   resistance. The pelvis is a common location for these fractures, which are usually seen, in elderly, osteoporotic
   women. In particular, the inferior and superior pubic rami are often affected, with a subgroup of these patients
   having avulsion insufficiency fractures of the ischial tuberosity. In this case, the most common presentation is
   sciatica due to irritation of the nearby sciatic nerve. For this reason, diagnosis may be delayed as the potential
   for lumbar spine pathology is evaluated. These fractures are usually visible on radiography. In the very acute
   phase, non-displaced fractures may be difficult to identify. Scintigraphy is very sensitive but lacks specificity
   and anatomic resolution. MRI will show the edema and any associated tendon or muscle injuries. CT will best
   display the fracture, and is particularly helpful in excluding pathologic fractures in the sacrum. In our case, the
   inferior pubic rami are well seen and normal bilaterally, excluding the possibility of acute or subacute fracture.
C) Incorrect. The adductor muscle group includes the adductor magnus, brevis and longus as well as the gracilis,
   pectineus and Sartorious muscles. These muscles principally take their origin from the pubic ramus and are
   located in the medial thigh, primarily acting to abduct the thigh although individual muscles in this group
   contribute to actions such as hip flexion and extension. Muscle injuries in general may be divided into
   contusions or strains, partial tears and complete tears or lacerations. Radiography is usually normal, but will
   show associated osseous injuries. MRI is the preferred imaging modality for evaluating muscle injuries. A
   muscle contusion or strain will appear as an intact muscle with increased T2 signal suggesting edema. Partial or
   complete tears can also be identified. In the acute phase, MRI may show hemorrhage at the site of injury. In
   the image shown for this question, some of the upper adductor musculature is visible, notably the adductor
   magnus and brevis. These muscles are entirely normal in their signal characteristics and morphology with no
   edema or loss of muscle bulk, effectively excluding a significant muscle injury.
D) Incorrect. Myositis is a non-specific term indicating inflammation within a muscle. Etiologies include bacterial,
   viral, and parasitic infections, collagen vascular diseases such as SLE and even drug toxicities. Pyomyositis is a
   distinct entity, which is often related to staphylococcal infection. The imaging appearance of myositis is also
   non-specific. On MRI, muscles may have increased T2 signal and there may be loss of distinction between
   tissue planes on T1 weighted sequences. Contrast-enhanced studies, using CT or MRI, will show any
   associated abscesses such as those seen with pyogenic infections. Accurate diagnosis requires aspiration and
   culture of the recovered material. Depending on the location, ultrasound or CT can be used for imaged guided
   aspiration or drain placements. In our case, all of the muscles, including the gluteus group, are normal with no
   bright T2 signal except at the sight of the tendon avulsion, making myositis an extremely unlikely diagnosis in
   this example.


                                    Diagnostic In-Training Exam 2003                                                     5
Section III – Musculoskeletal Radiology
E) Incorrect. Denervation injury to muscle can be the result of acute or chronic trauma to a nerve or other processes
   such as inflammatory neuropathies. Compressive neuropathies such as the anterior interosseous nerve syndrome
   or carpal tunnel syndrome also fall into this disease category. Initial imaging findings may be negative despite
   positive clinical examinations or studies such as EMG. As the disease progresses, the muscles that are innervated
   by the affected nerve may show some mild increased T2 signal. As the disease becomes chronic, there is often
   loss of muscle bulk and fatty replacement. In the case of large peripheral nerves such as the sciatic or median
   nerves, the abnormality in the nerve itself may be seen as increased size and T2 signal. In the case of smaller
   nerves, which cannot be easily resolved on imaging studies, the key to diagnosis is recognizing the pattern of
   muscle involvement relating to a specific nerve. In our case, the muscles are normal and even though there is
   some fluid surrounding the right sciatic nerve, the nerve itself is normal and symmetric with the opposite side.

Citations:
Brandser EA, el-Khoury GY, Kathol MH, Callaghan JJ, Tearse DS. Hamstring injuries: radiographic, conventional
    tomographic, CT, and MR imaging characteristics. Radiology 197(1): 257-62, 1995.




6                                      American College of Radiology
Section III – Musculoskeletal Radiology




                   Figure 3A                                                 Figure 3B


92.   You are shown PA (Figure 3A) and oblique (Figure 3B) radiographs of the hand in a 40-year-old woman with
      hand pain. Which one of the following is the MOST likely diagnosis?
      A. Gout
      B.   Systemic Lupus Erythematosus
      C. Erosive osteoarthritis
      D. Scleroderma
      E.   Rheumatoid arthritis




                                  Diagnostic In-Training Exam 2003                                           7
Section III – Musculoskeletal Radiology
Question #92
Findings: PA and oblique radiographs of the hand show diffusely decreased bone density in this 40-year-old woman.
On the oblique view, there are multiple subluxed MCP and IP joints with dislocation of the 5th PIP joint on the
oblique image with near complete reduction on the PA view. No erosions or productive changes are seen.

Rationales:
A) Incorrect. Gout is a crystalline deposition disease. Both primary and secondary gout are related to hyperuricemia,
   with the primary form representing an inborn error of metabolism and secondary gout arising from altered uric
   acid metabolism associated with other clinical disorders. Primary or idiopathic gout is much more common in
   men and typically presents in the fifth decade. It often begins as a monoarticular or oligoarticular disorder,
   progressing to involve more joints over time. The most commonly involved joint is the first metatarsophalangeal
   joint, which is, altered 75-100% of patients with gout. Most patients with biochemical and clinical evidence of
   gout will not have bone changes due to early treatment of the metabolic disorder. In patients who do manifest
   radiographic changes, the findings are usually classic and diagnostic of gout. The joint space is well preserved,
   with erosions in intra-articular and para-articular locations. These erosions are well defined with sclerotic margins
   and overhanging edges in up to 40%. The overhanging edges may be associated with gouty tophi in the adjacent
   soft tissues, which often show increased density or even calcification on x-rays. Bone density is usually preserved.
   The distribution in the skeleton is typical, with the feet most commonly involved. Hand and wrist, the knee and
   elbow are also usual sites. Cross sectional imaging is rarely contributory in the evaluation of gout. MRI may be
   helpful in early gout to identify synovitis and early erosions as well as the extent of soft tissue involvement. Gout
   would not be an appropriate consideration in the case in question. There are no erosions or other destructive
   changes present, effectively excluding a radiographic diagnosis of gout.
B) Correct. Systemic lupus erythematosus is an autoimmune connective tissue disorder affecting multiple organ
   systems. In the musculoskeletal system, common manifestations are a deforming, symmetric polyarthritis,
   myositis, tendon weakening, rupture, and osteonecrosis. As with most collagen vascular diseases, adult women
   are most commonly affected. The hallmark of the arthritis associated with SLE is deformity without destruction.
   The small joints of the hand are characteristically involved with multiple subluxed or even dislocated joints,
   which are easily and usually completely reducible. In fact, the positioning of the patient for the PA radiograph
   of the hand may itself reduce the subluxations making the disease less prominent. A relaxed oblique or “ball-
   catcher’s” view often shows the subluxations to better advantage as in our case. Periarticular osteopenia is
   common and reminiscent of rheumatoid arthritis, but the complete lack of any erosive changes should help to
   distinguish these entities. Osteonecrosis may be present in the form of avascular necrosis, bone infarcts, or both
   but this usually involves long bones rather than the small bones of the hands. Jacoud’s arthropathy, a sequela of
   rheumatic fever, has an appearance that is identical radiographically to SLE. Fortunately, this has become very
   rare in the era of antibiotic treatment of streptococcal infections, limiting differential diagnostic considerations.
C) Incorrect. Erosive osteoarthritis is an inflammatory variant of osteoarthritis, which is characterized by a
   combination of erosive and productive changes typically in the DIP joints of the hands. This disease usually
   affects middle-aged women. While other changes of degenerative osteoarthritis may be present within the
   other joints of the hand and wrist, the DIP involvement is usually strikingly worse. The primary differential
   considerations are other types of inflammatory arthritis, including psoriatic arthritis, Reiter’s syndrome, and
   even metabolic disorders such as hyperparathyroidism. What distinguish erosive osteoarthritis from these entities
   are the distribution of the radiographic changes and the lack of associated systemic disease. Psoriatic arthritis
   may affect women in the same age group. This is usually not confined to the DIP joints and the pattern of
   erosive changes is different. Reiter’s syndrome usually affects younger men and more typically involves the lower
   extremities. Rheumatoid arthritis spares the DIP joints in nearly all cases. Other forms of inflammatory and
   erosive arthritis also have typical radiographic patterns of disease. In our case, the complete lack of any erosive
   or productive changes in the interphalangeal joints excludes erosive osteoarthritis as a possibility.




8                                       American College of Radiology
Section III – Musculoskeletal Radiology
D) Incorrect. Scleroderma, or progressive systemic sclerosis, is a rare disorder of connective tissue affecting multiple
   organ systems most commonly affecting women in the third to fifth decades. In the musculoskeletal system,
   the hands are the most common sites of involvement, with changes most pronounced in the digits. Progressive
   atrophy of the soft tissues at the tips of the fingers creates a characteristically conical appearance to the finger.
   Progressive erosion and resorption of the ungual tuft of the fingers is also commonly seen with amorphous
   calcifications seen in the soft tissues. With time, the more proximal bones of the fingers may be involved as well
   with further resorption giving a “pencil” appearance to the digit. The feet are usually not affected to the same
   degree as the hands. Other sites where bony changes can be seen include the ribs, spine, and mandible. Soft
   tissue calcifications often are more diffuse and periarticular tumoral calcinosis may be seen. In the index case,
   the bones of fingers are normal except for their alignment and no soft tissue resorption or calcifications are seen.
E) Incorrect. Rheumatoid arthritis is a relatively common inflammatory arthritis affecting synovial joints, bursae,
   and tendon sheaths. The primary process is one of synovial inflammation with secondary affects on the
   underlying bone and cartilage. The small joints of the hands are commonly involved as are the cervical spine,
   feet and other sites. In the early stages of disease the primary finding may be soft tissue swelling over the MCP
   joints and ulnar styloid representing synovitis. Periarticular osteopenia may also be present early on reflecting
   hyperemia at the inflamed joints. With progression, erosions are seen in characteristic locations such as the
   ulnar styloid and MCP joints but no productive changes are present. The DIP joints are uniformly spared. As
   the inflamed synovium destroys ligaments and tendons, subluxations such as the swan’s neck deformity become
   common. Although the subluxations in our case could be seen in rheumatoid arthritis, the complete lack of any
   destructive changes or soft tissue swelling makes this much less likely.

Citations:
Brower A, Flemming DJ, Bralow L. Arthritis in Black and White. 2cd edition, W.B. Saunders, Philadelphia, 1997.




                                    Diagnostic In-Training Exam 2003                                                 9
Section III – Musculoskeletal Radiology




                    Figure 4A                                                   Figure 4B


93.   A 45-year-old man presents with ankle pain after playing basketball. You are shown sagittal T1 (Figure 4A)
      and T2 and sagittal-T2 (Figure 4B) weighted MR images of the ankle. Which one of the following is the
      most likely diagnosis?
      A. Pilon fracture
      B.   Achilles tenosynovitis
      C. Achilles tendon rupture
      D. Os trigonum syndrome
      E.   Calcaneal stress fracture




10                                     American College of Radiology
Section III – Musculoskeletal Radiology
Question #93
Findings: Sagittal T1 and fat suppressed T2 weighted MR images of the ankle show disruption of the Achilles
tendon with retraction of several centimeters. The visualized distal tendon is thickened with increased intra substance
signal. There is increased T2 signal in the gap and the adjacent soft tissues. A small ankle effusion is present. Osseous
structures are normal.

Rationales:
A) Incorrect. Pilon fractures of the ankle result from pronation-dorsiflexion injuries, which drive the talar dome
   into the tibial plafond. Included in this complex are an oblique fracture of the medial malleolus and an
   intraarticular fracture of the distal tibia, which may have more than one part. A fibular fracture may also be
   present. These are uncommon fractures, representing less than 0.5% of all ankle fractures. Treatment decisions
   are based on the degree of comminution of the fracture and the extent of intraarticular involvement and
   displacement. Radiography and knowledge of the mechanism of injury are usually diagnostic, but CT scanning
   with multiplanar reformatting may be helpful to assess f small intra-articular fragments and to better quantify
   the degree of displacement and articular incongruence. MRI is generally not contributory to management in
   the acute phase. In our case, the MRI of the ankle shows a normal tibia and talar dome, excluding fracture from
   the diagnosis.
B) Incorrect. The Achilles tendon is the largest tendon in the body. It represents the confluence of the
   gastrocnemius and soleus tendons, inserting on the posterior calcaneus. In adults, the Achilles tendon is
   approximately 10-15cm long. Unique among the tendons of the ankle, the Achilles tendon does not have a
   synovial-lined tendon sheath, so that tenosynovitis involving the Achilles tendon is not a possibility. Rather, it is
   covered by a peritenon. Peri- or para tendonitis may be seen within the surrounding soft tissues, often associated
   with tendinoplasty or partial tears of the tendon itself. Retrocalcaneal bursitis may also be seen just anterior to
   the insertion of the Achilles tendon on the calcaneus. Our case shows the clear discontinuity of the Achilles
   tendon with surrounding fluid related to the injury, but is not suggestion of a separate inflammatory process.
C) Correct. Achilles tendon rupture is most common in men between the ages of 30 and 50. The typical scenario
   is a “weekend warrior” who participates in a sport such as basketball, which uses sudden, forceful dorsiflexion or
   push off of the foot. Clinically there is sudden onset of pain and soft tissue swelling with an inability to stand on
   tiptoe on the affected side. Radiographs are usually obtained to exclude fracture and may show loss of the soft
   tissue planes surrounding the Achilles tendon or may even suggest disruption and retraction of the tendon itself.
   MRI definitively shows the disruption as well as its location and the degree of retraction. The most common site
   of a complete tear is approximately 2-6 cm proximal to the insertion of the tendon on the calcaneus. This site is
   vulnerable both to partial and complete tears as a relatively avascular portion of the tendon. In addition to
   trauma in unconditioned individuals, Achilles tendon rupture may be associated with chronic tendinoplasty and
   partial tears of the tendon, rheumatoid arthritis, SLE, and the use of local or systemic corticosteroids. MRI may
   show hemorrhage or fluid in the acute phase as well as the discontinuity of the tendon. When the problem is
   subacute or chronic, as in our case, the MRI shows discontinuity and some mild retraction as well as a thickened
   Achilles tendon with increased signal in the distal portion due to underlying tendinoplasty. Increased T2 signal
   is seen surrounding the ruptured tendon consistent with edema and fluid.
D) Incorrect. The Os Trigonum or talar compression syndrome is a pain syndrome involving the posterior ankle.
   The os trigonum is an accessory ossicle just posterior to the talus at the ankle. When fused to the posterior talus
   it is referred to as Stieda’s process. The flexor hallucis longus (FHL) tendon lies immediately adjacent to the os
   trigonum. In some cases where the os trigonum is enlarged or is relatively more mobile, irritation of the FHL
   tendon or the posterior talus itself may be seen, with tenosynovitis of the FHL or even partial tears of the tendon
   resulting. Radiography will show the enlarged ossicle, which may appear irregular, but MRI is diagnostic,
   showing the edema surrounding the os trigonum and posterior talus and the associated abnormalities in the
   FHL tendon. In our case, the posterior talus is normal with no edema and the FHL tendon is not included on
   the images shown.



                                    Diagnostic In-Training Exam 2003                                                11
Section III – Musculoskeletal Radiology
E) Incorrect. Stress fractures are the result of repetitive loading on bone, which may be normal or abnormal.
   Fatigue fractures involving normal bone may be due to a novel, strenuous activity, which places repetitive stress
   on a specific bone; an example would be the march fractures of the metatarsal seen in new military recruits. In
   the case of abnormal underlying bone, such as osteoporosis, the term insufficiency fracture may be applied.
   Common sites for insufficiency fractures include the pelvis and calcaneus. In the calcaneus, radiographs show
   the underlying osteopenia with crescentic area of sclerosis usually in the posterior calcaneus. The sclerosis
   probably represents a combination of impaction and healing. In the very acute phase, these fractures may be
   radiographically occult. In this case, MRI would show the reactive edema on T2 weighted imaging as well as
   the fracture itself, which appears as a linear area of low signal intensity on T1 weighted images.
     Scintigraphy and MRI have similar sensitivity, but MR is significantly more specific. CT with multiplanar
     reformatting will also show the fracture before it is radiographically evident. In our case, the calcaneus is
     normal on both T1 and T2 weighted images with normal marrow signal throughout and no edema to suggest
     fracture.

Citations:
Schepsis AA, Jones H, Haas AL. Achilles tendon disorders in athletes. American Journal of Sports Medicine 30(2):
    287-305, 2002.
Movin T, Kristoffersen-Wiberg M, Rolf C, Aspelin P. MR Imaging in chronic Achilles tendon disorder. Acta
   Radiologica 39(2): 126-32, 1998.




12                                     American College of Radiology
Section III – Musculoskeletal Radiology




                   Figure 5A                                                    Figure 5B




                                                 Figure 5C

94.   This 50-year-old man presented with a mass along his right chest wall. You are shown coronal T1 (Figure 5A),
      and fat-saturated post gadolinium T1 (Figure 5B) weighted images and a non-contrast axial CT scan (Figure
      5C) obtained 4 weeks after the MRI. Which one of the following is the MOST likely diagnosis?
      A. Myositis ossificans
      B.   Malignant fibrous histiocytoma
      C. Desmoid tumor
      D. Chondrosarcoma
      E.   Intramuscular myxoma

                                  Diagnostic In-Training Exam 2003                                            13
Section III – Musculoskeletal Radiology
Question #93
Findings: Coronal T1 and fat suppressed T1 post-gadolinium MR images of the right chest wall show a mass
within the lateral chest wall, which is isointense to muscle on the non-contrast examination and shows avid
enhancement with intravenous gadolinium. A small central area in the mass does not enhance and may be necrotic
or cystic. A single axial non-contrast CT image obtained four weeks later shows benign, peripheral calcification
within the teres major muscle. There has been no interval change in the size of the mass.

Rationales:
A) Correct. Myositis Ossificans (MO) is a localized soft tissue calcification most often related to antecedent injury.
   Up to 75% of patients who develop MO can relate a clear history of trauma to the affected area. Other causes
   may include burns, neurologic conditions, and systemic disorders associated with soft tissue calcification.
   Rarely the lesion may develop spontaneously. Common sites of MO include areas exposed to injury such as
   the buttocks, elbow, thigh, and calf. A soft tissue mass appears at the site of injury within about 10 days, with
   calcification usually appearing by 6 weeks. The calcification typically forms at the periphery of the soft tissue
   lesion, forming a smooth, complete border when mature. The peripheral calcification is an important feature
   distinguishing developing MO from malignancies such as osteosarcoma or chondrosarcoma. If the lesion is
   imaged with MR early in the process, the appearance can be very worrisome with avid contrast enhancement
   and features consistent with a soft tissue sarcoma such as a malignant fibrous histiocytoma. Even biopsy may
   be misleading if obtained from the cellular central portions of the mass. As with imaging, the more mature
   peripheral zone shows the benign nature of the lesion. Radiography will often show the mature peripheral
   calcification well enough for diagnostic confidence. If there is any question regarding the nature of the
   calcification (with an appropriate clinical history, of course) CT can be obtained which will show the benign
   calcification to best advantage. In our case, the patient was involved in a car accident with an injury to his chest
   wall approximately two weeks prior to the MRI. The MRI was felt to be worrisome for malignancy, but because
   of the history of trauma, biopsy was not immediately performed. CT obtained 4 weeks later shows the mature
   calcification of MO and no further evaluation was required.
B) Incorrect. Malignant fibrous histiocytoma (MFH) is the most common soft tissue sarcoma in adults. Rarely
   (5%) it may arise as a primary bone tumor. MFH presents as a painless soft tissue mass. Radiographs may show
   the tumor in the soft tissue if large, but the preferred modality for evaluation of soft tissue tumors is MRI.
   On MRI, the mass will be low in signal intensity on T1 weighted imaging and bright on T2. If contrast is
   administered, there is usually avid and homogeneous enhancement. In very large tumors there may be non-
   enhancing areas centrally representing necrosis. MRI best demonstrates the anatomic compartments involved
   and the relationship of the tumor to neurovascular structures, bones and joints. CT with contrast may be used
   if the patient is unable to undergo MRI. Ultrasound may be helpful for image-guided biopsy. Imaging is non-
   specific; high-grade liposarcomas and other types of primary sarcomas and metastases may have similar imaging
   features. MFH may rarely calcify, with the calcification located within the mass and appearing irregular. If the
   mass is adjacent to a bone, that bone may undergo pressure erosion. MFH was a consideration when the MRI
   was obtained in our patient, but because of the history of trauma, biopsy was deferred in favor of a follow-up
   CT.
C) Incorrect. Desmoid tumor is the name given to a benign fibrous proliferation arising in the abdominal and
   extra-abdominal musculature, often growing slowly and engulfing the surrounding tissue in an insidious fashion.
   While histologically benign, these tumors commonly recur after excision and are difficult to treat. As with most
   of the fibromatoses, they are locally aggressive and may cause erosion of adjacent bones with a periosteal reaction
   sometimes seen. They are usually solitary with a predilection for the shoulder girdle. Radiography may show the
   soft tissue mass if large enough as well as any effects on adjacent bone, but MRI is the preferred modality for
   diagnosis, with the dense fibrous tissue having characteristic signal intensity which is low on both T1 and T2
   weighted imaging. If intravenous gadolinium is given, there is usually marked enhancement throughout the
   lesion. Calcifications may be seen but are rare. In our case, the signal intensity of the lesion on the unenhanced
   T1 image is not as low as would be expected with a desmoid tumor. The post-gadolinium image shows the mass


14                                      American College of Radiology
Section III – Musculoskeletal Radiology
    to be well defined; desmoid tumors often have a less well-defined, more infiltrative appearance. The CT showing
    peripheral calcification in the mass is diagnostic of myositis ossificans and would be extremely unusual for a
    desmoid. The combination of history and the CT appearance effectively excludes the diagnosis of a desmoid
    tumor.
D) Incorrect. Chondrosarcomas are tumors of cartilaginous origin, which may occur primarily, or secondary to
   pre-existing lesions such as an enchondroma or osteochondroma. They may also be categorized according to
   their location in bone or by their histologic characteristics. Extraskeletal chondrosarcomas are rare. The most
   common location for chondrosarcoma is in the long tubular bones, with the femur being the most frequently
   affected bone. They are usually located in the metaphysis but extension to the epiphysis can be seen. Most
   chondrosarcomas are of low histologic grade. Radiographically, these tumors may show a primarily lytic area
   with endosteal scalloping and chondroid calcification within the lesion. Cortical thickening may also be seen.
   Higher grade or dedifferentiated chondrosarcomas will have a more aggressive appearance, with cortical
   breakthrough and large soft tissue masses. Radiography is the primary diagnostic modality. When the flat
   bones such as the pelvis and scapula are involved, CT may be preferred both to define the extent of the lesion
   and to characterize the calcifications. MRI may also be used to evaluate the extent of soft tissue and marrow
   involvement and to establish which anatomic compartments are involved for surgical planning. In the unusual
   case of an extraskeletal chondrosarcoma, a soft tissue mass with c chondroid calcification is seen. Our case shows
   smooth peripheral ossification, making a cartilaginous process extremely unlikely.
E) Incorrect. Intramuscular myxoma is an uncommon benign soft tissue mass. As its name would suggest, the mass
   is located within a muscle, most often in an extremity. Radiography rarely shows these lesions. MRI will show
   a well-circumscribed mass within a muscle, which is very low in signal intensity on T1, and very bright on T2
   weighted images. Myxomas rarely enhance except for a smooth, peripheral rim. These lesions almost never
   calcify. In our case, although the mass is located within the chest wall musculature, the mass enhances
   homogeneously which would be inconsistent with a myxoma, as would be the peripheral calcification.

Citations:
Jepsen MC, Graham SM. Traumatic myositis ossificans of the levator scapulae muscle, American Journal of
    Otolaryngology 19(5): 345-348, 1998.
Resnick, D. “Soft tissues” in Resnick, D. Bone and Joint Imaging, 2cd edition, 1261-1263, W.B. Saunders,
    Philadelphia, 1996.




                                   Diagnostic In-Training Exam 2003                                             15

More Related Content

What's hot

Msk Lecture3 1st Hospital
Msk Lecture3 1st HospitalMsk Lecture3 1st Hospital
Msk Lecture3 1st HospitalSumit Prajapati
 
Introduction skeletal radiology(11月20.)
Introduction   skeletal radiology(11月20.)Introduction   skeletal radiology(11月20.)
Introduction skeletal radiology(11月20.)ghalan
 
Presentation1.pptx, radiological imaging of soft tissue masses of the hand an...
Presentation1.pptx, radiological imaging of soft tissue masses of the hand an...Presentation1.pptx, radiological imaging of soft tissue masses of the hand an...
Presentation1.pptx, radiological imaging of soft tissue masses of the hand an...Abdellah Nazeer
 
Diagnostic Imaging of Spondyloarthropathies
Diagnostic Imaging of SpondyloarthropathiesDiagnostic Imaging of Spondyloarthropathies
Diagnostic Imaging of SpondyloarthropathiesMohamed M.A. Zaitoun
 
Presentation1.pptx, radiological imaging of rediolucent lesions of bones.
Presentation1.pptx, radiological imaging of rediolucent lesions of bones.Presentation1.pptx, radiological imaging of rediolucent lesions of bones.
Presentation1.pptx, radiological imaging of rediolucent lesions of bones.Abdellah Nazeer
 
Presentation1.pptx, lecture for md oral examination.
Presentation1.pptx, lecture for md oral examination.Presentation1.pptx, lecture for md oral examination.
Presentation1.pptx, lecture for md oral examination.Abdellah Nazeer
 
Spinal cord lesions and its radiological imaging finding.
Spinal cord lesions and its radiological imaging finding.Spinal cord lesions and its radiological imaging finding.
Spinal cord lesions and its radiological imaging finding.Navneet Ranjan
 
23204928
2320492823204928
23204928radgirl
 
Benign bone tumors imaging
Benign bone tumors imagingBenign bone tumors imaging
Benign bone tumors imagingRiyaz Ahmed
 
Painful bone metastases in adults
Painful bone metastases in adultsPainful bone metastases in adults
Painful bone metastases in adultsRamin Sadeghi
 
Bone tumor staging systems
Bone tumor staging systemsBone tumor staging systems
Bone tumor staging systemsAbdulla Kamal
 
Bone tumors ug
Bone tumors ugBone tumors ug
Bone tumors ugvichand8
 
Presentation1.pptx, radiological imaging of paget disease.
Presentation1.pptx, radiological imaging of paget disease.Presentation1.pptx, radiological imaging of paget disease.
Presentation1.pptx, radiological imaging of paget disease.Abdellah Nazeer
 
1362396733 imaging in diabetic foot
1362396733 imaging in diabetic foot1362396733 imaging in diabetic foot
1362396733 imaging in diabetic footdfsimedia
 
Presentation1.pptx, radiological imaging of spinal cord tumour.
Presentation1.pptx, radiological imaging of spinal cord tumour.Presentation1.pptx, radiological imaging of spinal cord tumour.
Presentation1.pptx, radiological imaging of spinal cord tumour.Abdellah Nazeer
 

What's hot (20)

Bone Tumors Benign Ppt
Bone Tumors Benign PptBone Tumors Benign Ppt
Bone Tumors Benign Ppt
 
Msk Lecture3 1st Hospital
Msk Lecture3 1st HospitalMsk Lecture3 1st Hospital
Msk Lecture3 1st Hospital
 
Introduction skeletal radiology(11月20.)
Introduction   skeletal radiology(11月20.)Introduction   skeletal radiology(11月20.)
Introduction skeletal radiology(11月20.)
 
Presentation1.pptx, radiological imaging of soft tissue masses of the hand an...
Presentation1.pptx, radiological imaging of soft tissue masses of the hand an...Presentation1.pptx, radiological imaging of soft tissue masses of the hand an...
Presentation1.pptx, radiological imaging of soft tissue masses of the hand an...
 
Diagnostic Imaging of Spondyloarthropathies
Diagnostic Imaging of SpondyloarthropathiesDiagnostic Imaging of Spondyloarthropathies
Diagnostic Imaging of Spondyloarthropathies
 
Diagnostic Ultrasound soft tissue
Diagnostic Ultrasound soft tissueDiagnostic Ultrasound soft tissue
Diagnostic Ultrasound soft tissue
 
Presentation1.pptx, radiological imaging of rediolucent lesions of bones.
Presentation1.pptx, radiological imaging of rediolucent lesions of bones.Presentation1.pptx, radiological imaging of rediolucent lesions of bones.
Presentation1.pptx, radiological imaging of rediolucent lesions of bones.
 
Presentation1.pptx, lecture for md oral examination.
Presentation1.pptx, lecture for md oral examination.Presentation1.pptx, lecture for md oral examination.
Presentation1.pptx, lecture for md oral examination.
 
Spinal cord lesions and its radiological imaging finding.
Spinal cord lesions and its radiological imaging finding.Spinal cord lesions and its radiological imaging finding.
Spinal cord lesions and its radiological imaging finding.
 
23204928
2320492823204928
23204928
 
Benign bone tumors imaging
Benign bone tumors imagingBenign bone tumors imaging
Benign bone tumors imaging
 
Painful bone metastases in adults
Painful bone metastases in adultsPainful bone metastases in adults
Painful bone metastases in adults
 
Bone tumor staging systems
Bone tumor staging systemsBone tumor staging systems
Bone tumor staging systems
 
Bone tumors ug
Bone tumors ugBone tumors ug
Bone tumors ug
 
Presentation1.pptx, radiological imaging of paget disease.
Presentation1.pptx, radiological imaging of paget disease.Presentation1.pptx, radiological imaging of paget disease.
Presentation1.pptx, radiological imaging of paget disease.
 
1362396733 imaging in diabetic foot
1362396733 imaging in diabetic foot1362396733 imaging in diabetic foot
1362396733 imaging in diabetic foot
 
Presentation1.pptx, radiological imaging of spinal cord tumour.
Presentation1.pptx, radiological imaging of spinal cord tumour.Presentation1.pptx, radiological imaging of spinal cord tumour.
Presentation1.pptx, radiological imaging of spinal cord tumour.
 
Bone Tumors
Bone TumorsBone Tumors
Bone Tumors
 
Bone tumors pre management
Bone tumors pre managementBone tumors pre management
Bone tumors pre management
 
Benign bone tumors
Benign bone tumorsBenign bone tumors
Benign bone tumors
 

Viewers also liked

الدوريات الأجنبية فى مكتبات الكليات العلمية فى جامعة أسيوط
الدوريات الأجنبية فى مكتبات الكليات العلمية فى جامعة أسيوطالدوريات الأجنبية فى مكتبات الكليات العلمية فى جامعة أسيوط
الدوريات الأجنبية فى مكتبات الكليات العلمية فى جامعة أسيوطEssam Obaid
 
Networking And Network Etiquette
Networking And Network EtiquetteNetworking And Network Etiquette
Networking And Network EtiquetteJMULLINMBA
 
erase asap
erase asaperase asap
erase asapricketki
 
Rootstech-The Basics of Gamification
Rootstech-The Basics of GamificationRootstech-The Basics of Gamification
Rootstech-The Basics of GamificationDave McAllister
 
Christmas Newsletter
Christmas NewsletterChristmas Newsletter
Christmas Newsletterandy biggin
 
Presentation Faculty Club with you tube video
Presentation Faculty Club with you tube videoPresentation Faculty Club with you tube video
Presentation Faculty Club with you tube videoMaikelVandeVelde
 
Health - Diagnostic and therapeutic tools
Health - Diagnostic and therapeutic toolsHealth - Diagnostic and therapeutic tools
Health - Diagnostic and therapeutic toolsstjulians school
 
Vakfotografie Sonny Lips
Vakfotografie Sonny LipsVakfotografie Sonny Lips
Vakfotografie Sonny LipsSonny1967
 
Blueoceanstrategy Benny Hersanto
Blueoceanstrategy Benny HersantoBlueoceanstrategy Benny Hersanto
Blueoceanstrategy Benny Hersantoguest03ecb9a
 
Retail Make Over Atcan Self Storage
Retail Make Over Atcan Self StorageRetail Make Over Atcan Self Storage
Retail Make Over Atcan Self StorageJim Emilson
 
Transit of Venus Presentation
Transit of Venus PresentationTransit of Venus Presentation
Transit of Venus Presentationbecnicholas
 
2009 Directors Report
2009 Directors Report2009 Directors Report
2009 Directors Reportandy biggin
 
23205029
2320502923205029
23205029radgirl
 
"5 Things in 5 Minutes" Series No.4 - "Mr. Banker, 5-Reasons Why the Bank Nee...
"5 Things in 5 Minutes" Series No.4 - "Mr. Banker, 5-Reasons Why the Bank Nee..."5 Things in 5 Minutes" Series No.4 - "Mr. Banker, 5-Reasons Why the Bank Nee...
"5 Things in 5 Minutes" Series No.4 - "Mr. Banker, 5-Reasons Why the Bank Nee...Arun Cavale Cavale
 
Application problems - Answers
Application problems - AnswersApplication problems - Answers
Application problems - AnswersCarlos Vázquez
 

Viewers also liked (20)

الدوريات الأجنبية فى مكتبات الكليات العلمية فى جامعة أسيوط
الدوريات الأجنبية فى مكتبات الكليات العلمية فى جامعة أسيوطالدوريات الأجنبية فى مكتبات الكليات العلمية فى جامعة أسيوط
الدوريات الأجنبية فى مكتبات الكليات العلمية فى جامعة أسيوط
 
Networking And Network Etiquette
Networking And Network EtiquetteNetworking And Network Etiquette
Networking And Network Etiquette
 
erase asap
erase asaperase asap
erase asap
 
Rootstech-The Basics of Gamification
Rootstech-The Basics of GamificationRootstech-The Basics of Gamification
Rootstech-The Basics of Gamification
 
EdTechProgramNationalUniversity
EdTechProgramNationalUniversityEdTechProgramNationalUniversity
EdTechProgramNationalUniversity
 
Christmas Newsletter
Christmas NewsletterChristmas Newsletter
Christmas Newsletter
 
Presentation Faculty Club with you tube video
Presentation Faculty Club with you tube videoPresentation Faculty Club with you tube video
Presentation Faculty Club with you tube video
 
Health - Diagnostic and therapeutic tools
Health - Diagnostic and therapeutic toolsHealth - Diagnostic and therapeutic tools
Health - Diagnostic and therapeutic tools
 
What’s new on f15
What’s new on f15What’s new on f15
What’s new on f15
 
Vakfotografie Sonny Lips
Vakfotografie Sonny LipsVakfotografie Sonny Lips
Vakfotografie Sonny Lips
 
Blueoceanstrategy Benny Hersanto
Blueoceanstrategy Benny HersantoBlueoceanstrategy Benny Hersanto
Blueoceanstrategy Benny Hersanto
 
Ever Bright Ventilation Training
Ever Bright Ventilation TrainingEver Bright Ventilation Training
Ever Bright Ventilation Training
 
Retail Make Over Atcan Self Storage
Retail Make Over Atcan Self StorageRetail Make Over Atcan Self Storage
Retail Make Over Atcan Self Storage
 
Transit of Venus Presentation
Transit of Venus PresentationTransit of Venus Presentation
Transit of Venus Presentation
 
2009 Directors Report
2009 Directors Report2009 Directors Report
2009 Directors Report
 
23205029
2320502923205029
23205029
 
"5 Things in 5 Minutes" Series No.4 - "Mr. Banker, 5-Reasons Why the Bank Nee...
"5 Things in 5 Minutes" Series No.4 - "Mr. Banker, 5-Reasons Why the Bank Nee..."5 Things in 5 Minutes" Series No.4 - "Mr. Banker, 5-Reasons Why the Bank Nee...
"5 Things in 5 Minutes" Series No.4 - "Mr. Banker, 5-Reasons Why the Bank Nee...
 
20090920 昆山ABC V4 Profile
20090920 昆山ABC V4 Profile20090920 昆山ABC V4 Profile
20090920 昆山ABC V4 Profile
 
Aaa! Newsletter
Aaa! NewsletterAaa! Newsletter
Aaa! Newsletter
 
Application problems - Answers
Application problems - AnswersApplication problems - Answers
Application problems - Answers
 

Similar to 23204952

Benign bone disease spotters
Benign bone disease spottersBenign bone disease spotters
Benign bone disease spottersAGRAWAL14
 
D. Firas lecture minimum muhadharaty require
D. Firas lecture minimum muhadharaty requireD. Firas lecture minimum muhadharaty require
D. Firas lecture minimum muhadharaty requirehussainAltaher
 
Dr.salah.radiology.bone diseases
Dr.salah.radiology.bone diseasesDr.salah.radiology.bone diseases
Dr.salah.radiology.bone diseasesabas_lb
 
Common benign and malignant bone tumors
Common benign and malignant bone tumorsCommon benign and malignant bone tumors
Common benign and malignant bone tumorsahm732
 
Benign Bone Tumors and Tumor Like Conditions
Benign Bone Tumors and Tumor Like Conditions Benign Bone Tumors and Tumor Like Conditions
Benign Bone Tumors and Tumor Like Conditions priyanka rana
 
23205008
2320500823205008
23205008radgirl
 
msk tumor.pdf
msk tumor.pdfmsk tumor.pdf
msk tumor.pdfZahra1373
 
Muculoskeletal Pediatic Imaging..pptx
Muculoskeletal Pediatic Imaging..pptxMuculoskeletal Pediatic Imaging..pptx
Muculoskeletal Pediatic Imaging..pptxAbdellah Nazeer
 
Pyogenic bone and joint infections
Pyogenic bone and joint infectionsPyogenic bone and joint infections
Pyogenic bone and joint infectionsBajanagaraju
 
Tumors of osseous origin
Tumors of osseous origin Tumors of osseous origin
Tumors of osseous origin Meeravali Shaik
 

Similar to 23204952 (20)

Bone tumors part 2
Bone tumors part 2Bone tumors part 2
Bone tumors part 2
 
Benign bone disease spotters
Benign bone disease spottersBenign bone disease spotters
Benign bone disease spotters
 
Radiology 5th year, 2nd lecture (Dr. Salah Mohammad Fatih)
Radiology 5th year, 2nd lecture (Dr. Salah Mohammad Fatih)Radiology 5th year, 2nd lecture (Dr. Salah Mohammad Fatih)
Radiology 5th year, 2nd lecture (Dr. Salah Mohammad Fatih)
 
D. Firas lecture minimum muhadharaty require
D. Firas lecture minimum muhadharaty requireD. Firas lecture minimum muhadharaty require
D. Firas lecture minimum muhadharaty require
 
Radiology 5th year, 1st lecture (Dr. Salah Mohammad Fatih)
Radiology 5th year, 1st lecture (Dr. Salah Mohammad Fatih)Radiology 5th year, 1st lecture (Dr. Salah Mohammad Fatih)
Radiology 5th year, 1st lecture (Dr. Salah Mohammad Fatih)
 
Sclerotic
ScleroticSclerotic
Sclerotic
 
Osteosarcoma (knee joint)
Osteosarcoma (knee joint)Osteosarcoma (knee joint)
Osteosarcoma (knee joint)
 
Dr.salah.radiology.bone diseases
Dr.salah.radiology.bone diseasesDr.salah.radiology.bone diseases
Dr.salah.radiology.bone diseases
 
OS image fin.pdf
OS image fin.pdfOS image fin.pdf
OS image fin.pdf
 
Common benign and malignant bone tumors
Common benign and malignant bone tumorsCommon benign and malignant bone tumors
Common benign and malignant bone tumors
 
International Journal of Orthopedics: Research & Therapy
International Journal of Orthopedics: Research & TherapyInternational Journal of Orthopedics: Research & Therapy
International Journal of Orthopedics: Research & Therapy
 
Benign Bone Tumors and Tumor Like Conditions
Benign Bone Tumors and Tumor Like Conditions Benign Bone Tumors and Tumor Like Conditions
Benign Bone Tumors and Tumor Like Conditions
 
23205008
2320500823205008
23205008
 
msk tumor.pdf
msk tumor.pdfmsk tumor.pdf
msk tumor.pdf
 
Muculoskeletal Pediatic Imaging..pptx
Muculoskeletal Pediatic Imaging..pptxMuculoskeletal Pediatic Imaging..pptx
Muculoskeletal Pediatic Imaging..pptx
 
Pyogenic bone and joint infections
Pyogenic bone and joint infectionsPyogenic bone and joint infections
Pyogenic bone and joint infections
 
Bone tumors
Bone tumorsBone tumors
Bone tumors
 
Osteosarcoma
Osteosarcoma Osteosarcoma
Osteosarcoma
 
Osteomylitis ppt
Osteomylitis pptOsteomylitis ppt
Osteomylitis ppt
 
Tumors of osseous origin
Tumors of osseous origin Tumors of osseous origin
Tumors of osseous origin
 

More from radgirl

23205065
2320506523205065
23205065radgirl
 
23205062
2320506223205062
23205062radgirl
 
23205059
2320505923205059
23205059radgirl
 
23205056
2320505623205056
23205056radgirl
 
23205042
2320504223205042
23205042radgirl
 
23205052
2320505223205052
23205052radgirl
 
23205045
2320504523205045
23205045radgirl
 
23205048
2320504823205048
23205048radgirl
 
23205039
2320503923205039
23205039radgirl
 
23205036
2320503623205036
23205036radgirl
 
23205032
2320503223205032
23205032radgirl
 
23205025
2320502523205025
23205025radgirl
 
23205022
2320502223205022
23205022radgirl
 
23205019
2320501923205019
23205019radgirl
 
23205016
2320501623205016
23205016radgirl
 
23205011
2320501123205011
23205011radgirl
 
23204998
2320499823204998
23204998radgirl
 
23205004
2320500423205004
23205004radgirl
 
23205001
2320500123205001
23205001radgirl
 
23204995
2320499523204995
23204995radgirl
 

More from radgirl (20)

23205065
2320506523205065
23205065
 
23205062
2320506223205062
23205062
 
23205059
2320505923205059
23205059
 
23205056
2320505623205056
23205056
 
23205042
2320504223205042
23205042
 
23205052
2320505223205052
23205052
 
23205045
2320504523205045
23205045
 
23205048
2320504823205048
23205048
 
23205039
2320503923205039
23205039
 
23205036
2320503623205036
23205036
 
23205032
2320503223205032
23205032
 
23205025
2320502523205025
23205025
 
23205022
2320502223205022
23205022
 
23205019
2320501923205019
23205019
 
23205016
2320501623205016
23205016
 
23205011
2320501123205011
23205011
 
23204998
2320499823204998
23204998
 
23205004
2320500423205004
23205004
 
23205001
2320500123205001
23205001
 
23204995
2320499523204995
23204995
 

Recently uploaded

Valproic Acid. (VPA). Antiseizure medication
Valproic Acid.  (VPA). Antiseizure medicationValproic Acid.  (VPA). Antiseizure medication
Valproic Acid. (VPA). Antiseizure medicationMohamadAlhes
 
Presentation for Bella Mahl 2024-03-28-24-MW-Overview-Bella.pptx
Presentation for Bella Mahl 2024-03-28-24-MW-Overview-Bella.pptxPresentation for Bella Mahl 2024-03-28-24-MW-Overview-Bella.pptx
Presentation for Bella Mahl 2024-03-28-24-MW-Overview-Bella.pptxpdamico1
 
COVID-19 (NOVEL CORONA VIRUS DISEASE PANDEMIC ).pptx
COVID-19  (NOVEL CORONA  VIRUS DISEASE PANDEMIC ).pptxCOVID-19  (NOVEL CORONA  VIRUS DISEASE PANDEMIC ).pptx
COVID-19 (NOVEL CORONA VIRUS DISEASE PANDEMIC ).pptxBibekananda shah
 
Myelin Oligodendrocyte Glycoprotein antibody associated disease (MOGAD)
Myelin Oligodendrocyte Glycoprotein antibody associated disease (MOGAD)Myelin Oligodendrocyte Glycoprotein antibody associated disease (MOGAD)
Myelin Oligodendrocyte Glycoprotein antibody associated disease (MOGAD)MohamadAlhes
 
Rheumatoid arthritis - Musculoskeletal disorders.ppt
Rheumatoid arthritis - Musculoskeletal disorders.pptRheumatoid arthritis - Musculoskeletal disorders.ppt
Rheumatoid arthritis - Musculoskeletal disorders.pptraviapr7
 
Chronic-Fatigue-Syndrome-CFS-Understanding-a-Complex-Disorder.pptx
Chronic-Fatigue-Syndrome-CFS-Understanding-a-Complex-Disorder.pptxChronic-Fatigue-Syndrome-CFS-Understanding-a-Complex-Disorder.pptx
Chronic-Fatigue-Syndrome-CFS-Understanding-a-Complex-Disorder.pptxSasikiranMarri
 
SYNDESMOTIC INJURY- ANATOMICAL REPAIR.pptx
SYNDESMOTIC INJURY- ANATOMICAL REPAIR.pptxSYNDESMOTIC INJURY- ANATOMICAL REPAIR.pptx
SYNDESMOTIC INJURY- ANATOMICAL REPAIR.pptxdrashraf369
 
Phytochemical Investigation of Drugs PDF.pdf
Phytochemical Investigation of Drugs PDF.pdfPhytochemical Investigation of Drugs PDF.pdf
Phytochemical Investigation of Drugs PDF.pdfDivya Kanojiya
 
Musculoskeletal disorders: Osteoarthritis,.pptx
Musculoskeletal disorders: Osteoarthritis,.pptxMusculoskeletal disorders: Osteoarthritis,.pptx
Musculoskeletal disorders: Osteoarthritis,.pptxraviapr7
 
Introduction to Sports Injuries by- Dr. Anjali Rai
Introduction to Sports Injuries by- Dr. Anjali RaiIntroduction to Sports Injuries by- Dr. Anjali Rai
Introduction to Sports Injuries by- Dr. Anjali RaiGoogle
 
Informed Consent Empowering Healthcare Decision-Making.pptx
Informed Consent Empowering Healthcare Decision-Making.pptxInformed Consent Empowering Healthcare Decision-Making.pptx
Informed Consent Empowering Healthcare Decision-Making.pptxSasikiranMarri
 
SHOCK (Medical SURGICAL BASED EDITION)).pptx
SHOCK (Medical SURGICAL BASED EDITION)).pptxSHOCK (Medical SURGICAL BASED EDITION)).pptx
SHOCK (Medical SURGICAL BASED EDITION)).pptxAbhishek943418
 
Primary headache and facial pain. (2024)
Primary headache and facial pain. (2024)Primary headache and facial pain. (2024)
Primary headache and facial pain. (2024)Mohamed Rizk Khodair
 
Big Data Analysis Suggests COVID Vaccination Increases Excess Mortality Of ...
Big Data Analysis Suggests COVID  Vaccination Increases Excess Mortality Of  ...Big Data Analysis Suggests COVID  Vaccination Increases Excess Mortality Of  ...
Big Data Analysis Suggests COVID Vaccination Increases Excess Mortality Of ...sdateam0
 
World-Health-Day-2024-My-Health-My-Right.pptx
World-Health-Day-2024-My-Health-My-Right.pptxWorld-Health-Day-2024-My-Health-My-Right.pptx
World-Health-Day-2024-My-Health-My-Right.pptxEx WHO/USAID
 
Systemic Lupus Erythematosus -SLE PT2.ppt
Systemic  Lupus  Erythematosus -SLE PT2.pptSystemic  Lupus  Erythematosus -SLE PT2.ppt
Systemic Lupus Erythematosus -SLE PT2.pptraviapr7
 
History and Development of Pharmacovigilence.pdf
History and Development of Pharmacovigilence.pdfHistory and Development of Pharmacovigilence.pdf
History and Development of Pharmacovigilence.pdfSasikiranMarri
 
Radiation Dosimetry Parameters and Isodose Curves.pptx
Radiation Dosimetry Parameters and Isodose Curves.pptxRadiation Dosimetry Parameters and Isodose Curves.pptx
Radiation Dosimetry Parameters and Isodose Curves.pptxDr. Dheeraj Kumar
 
Basic principles involved in the traditional systems of medicine PDF.pdf
Basic principles involved in the traditional systems of medicine PDF.pdfBasic principles involved in the traditional systems of medicine PDF.pdf
Basic principles involved in the traditional systems of medicine PDF.pdfDivya Kanojiya
 
ANEMIA IN PREGNANCY by Dr. Akebom Kidanemariam
ANEMIA IN PREGNANCY by Dr. Akebom KidanemariamANEMIA IN PREGNANCY by Dr. Akebom Kidanemariam
ANEMIA IN PREGNANCY by Dr. Akebom KidanemariamAkebom Gebremichael
 

Recently uploaded (20)

Valproic Acid. (VPA). Antiseizure medication
Valproic Acid.  (VPA). Antiseizure medicationValproic Acid.  (VPA). Antiseizure medication
Valproic Acid. (VPA). Antiseizure medication
 
Presentation for Bella Mahl 2024-03-28-24-MW-Overview-Bella.pptx
Presentation for Bella Mahl 2024-03-28-24-MW-Overview-Bella.pptxPresentation for Bella Mahl 2024-03-28-24-MW-Overview-Bella.pptx
Presentation for Bella Mahl 2024-03-28-24-MW-Overview-Bella.pptx
 
COVID-19 (NOVEL CORONA VIRUS DISEASE PANDEMIC ).pptx
COVID-19  (NOVEL CORONA  VIRUS DISEASE PANDEMIC ).pptxCOVID-19  (NOVEL CORONA  VIRUS DISEASE PANDEMIC ).pptx
COVID-19 (NOVEL CORONA VIRUS DISEASE PANDEMIC ).pptx
 
Myelin Oligodendrocyte Glycoprotein antibody associated disease (MOGAD)
Myelin Oligodendrocyte Glycoprotein antibody associated disease (MOGAD)Myelin Oligodendrocyte Glycoprotein antibody associated disease (MOGAD)
Myelin Oligodendrocyte Glycoprotein antibody associated disease (MOGAD)
 
Rheumatoid arthritis - Musculoskeletal disorders.ppt
Rheumatoid arthritis - Musculoskeletal disorders.pptRheumatoid arthritis - Musculoskeletal disorders.ppt
Rheumatoid arthritis - Musculoskeletal disorders.ppt
 
Chronic-Fatigue-Syndrome-CFS-Understanding-a-Complex-Disorder.pptx
Chronic-Fatigue-Syndrome-CFS-Understanding-a-Complex-Disorder.pptxChronic-Fatigue-Syndrome-CFS-Understanding-a-Complex-Disorder.pptx
Chronic-Fatigue-Syndrome-CFS-Understanding-a-Complex-Disorder.pptx
 
SYNDESMOTIC INJURY- ANATOMICAL REPAIR.pptx
SYNDESMOTIC INJURY- ANATOMICAL REPAIR.pptxSYNDESMOTIC INJURY- ANATOMICAL REPAIR.pptx
SYNDESMOTIC INJURY- ANATOMICAL REPAIR.pptx
 
Phytochemical Investigation of Drugs PDF.pdf
Phytochemical Investigation of Drugs PDF.pdfPhytochemical Investigation of Drugs PDF.pdf
Phytochemical Investigation of Drugs PDF.pdf
 
Musculoskeletal disorders: Osteoarthritis,.pptx
Musculoskeletal disorders: Osteoarthritis,.pptxMusculoskeletal disorders: Osteoarthritis,.pptx
Musculoskeletal disorders: Osteoarthritis,.pptx
 
Introduction to Sports Injuries by- Dr. Anjali Rai
Introduction to Sports Injuries by- Dr. Anjali RaiIntroduction to Sports Injuries by- Dr. Anjali Rai
Introduction to Sports Injuries by- Dr. Anjali Rai
 
Informed Consent Empowering Healthcare Decision-Making.pptx
Informed Consent Empowering Healthcare Decision-Making.pptxInformed Consent Empowering Healthcare Decision-Making.pptx
Informed Consent Empowering Healthcare Decision-Making.pptx
 
SHOCK (Medical SURGICAL BASED EDITION)).pptx
SHOCK (Medical SURGICAL BASED EDITION)).pptxSHOCK (Medical SURGICAL BASED EDITION)).pptx
SHOCK (Medical SURGICAL BASED EDITION)).pptx
 
Primary headache and facial pain. (2024)
Primary headache and facial pain. (2024)Primary headache and facial pain. (2024)
Primary headache and facial pain. (2024)
 
Big Data Analysis Suggests COVID Vaccination Increases Excess Mortality Of ...
Big Data Analysis Suggests COVID  Vaccination Increases Excess Mortality Of  ...Big Data Analysis Suggests COVID  Vaccination Increases Excess Mortality Of  ...
Big Data Analysis Suggests COVID Vaccination Increases Excess Mortality Of ...
 
World-Health-Day-2024-My-Health-My-Right.pptx
World-Health-Day-2024-My-Health-My-Right.pptxWorld-Health-Day-2024-My-Health-My-Right.pptx
World-Health-Day-2024-My-Health-My-Right.pptx
 
Systemic Lupus Erythematosus -SLE PT2.ppt
Systemic  Lupus  Erythematosus -SLE PT2.pptSystemic  Lupus  Erythematosus -SLE PT2.ppt
Systemic Lupus Erythematosus -SLE PT2.ppt
 
History and Development of Pharmacovigilence.pdf
History and Development of Pharmacovigilence.pdfHistory and Development of Pharmacovigilence.pdf
History and Development of Pharmacovigilence.pdf
 
Radiation Dosimetry Parameters and Isodose Curves.pptx
Radiation Dosimetry Parameters and Isodose Curves.pptxRadiation Dosimetry Parameters and Isodose Curves.pptx
Radiation Dosimetry Parameters and Isodose Curves.pptx
 
Basic principles involved in the traditional systems of medicine PDF.pdf
Basic principles involved in the traditional systems of medicine PDF.pdfBasic principles involved in the traditional systems of medicine PDF.pdf
Basic principles involved in the traditional systems of medicine PDF.pdf
 
ANEMIA IN PREGNANCY by Dr. Akebom Kidanemariam
ANEMIA IN PREGNANCY by Dr. Akebom KidanemariamANEMIA IN PREGNANCY by Dr. Akebom Kidanemariam
ANEMIA IN PREGNANCY by Dr. Akebom Kidanemariam
 

23204952

  • 1. Section III – Musculoskeletal Radiology Figure 1A Figure 1B 90. A 15-year-old woman presents with aching knee pain and no history of trauma. You are shown an AP radiograph (Figure 1A) and non-contrast CT image (Figure 1B). Which one of the following is the MOST likely diagnosis? A. Osteomyelitis B. Osteosarcoma C. Chondroblastoma D. Osteoid Osteoma E. Ewing’s Sarcoma Diagnostic In-Training Exam 2003 1
  • 2. Section III – Musculoskeletal Radiology Question #90 Findings: The standing AP radiograph of the femur shows a very dense but smooth and benign appearing periosteal reaction at the medial aspect of the distal femoral Meta diaphysis. No destructive or aggressive features are present on the image. The single axial, non-contrast CT image shows the dense reactive bone involving the medial and posterior cortex as well as a small lucent nidus with central calcification within the cortex itself. There is no soft tissue mass or any disruption of soft tissue planes on the CT image. Rationales: A) Incorrect. The pattern of osteomyelitis depends on the age of the patient and the mode of infection. In infants, perforating vessels cross the open growth plate and hematogenously spread infection can extend to the epiphysis. In childhood and early adolescence, those perforating vessels regress and there is not communication across the open growth plate and infection more commonly involves the metaphysis or Meta diaphysis. As the growth plate closes, there are again patent vessels, which allow communication between the metaphysis and epiphysis, but it is far more common for adults to acquire osteomyelitis from a source of infection in the contiguous soft tissues rather than from a hematogenous source. Radiographically, osteomyelitis usually presents as lucency in the bone. Depending on the infectious agent and the chronicity of infection, there may be more or less reactive bone near the lesion. With subacute or chronic infection, well-defined intraosseous abscesses with a sclerotic margin may develop (Brodie’s abscess) and sinus tracts may be seen. Bony sequestra, which are intracortical pieces of necrotic bone surrounded by granulation tissue, may also be seen. In the case of bone infection from an adjacent soft tissue source, the key diagnostic feature distinguishing osteomyelitis from reactive change to cellulitis is cortical destruction. In the early phases of infection, subtle changes in the bone may be imperceptible radiographically. MRI is both more sensitive and more specific for the bone changes in addition to identifying the accompanying soft tissue abnormalities. The specificity of MRI may be improved by the addition of nuclear medicine studies using labeled white cells to localize infection. In our case, the calcified nidus might possibly be taken for a small sequestrum and osteomyelitis is a differential diagnostic consideration here. However, the growth plate in this adolescent girl is closed, making the metaphysis an unlikely location for infection in this patient and the dense, benign reactive bone would be atypical for even the most chronic infection. B) Incorrect. Osteosarcoma is second only to myeloma in frequency as a primary malignancy of bone. Commonly presenting in the second and third decades, approximately 50-75% of the most common variant, conventional osteosarcoma, occurs about the knee. The tumor typically has an aggressive appearance, with destruction of the underlying bone and variable production of malignant appearing osteoid. Radiographically osteosarcoma is an ill-defined, destructive intramedullary, metaphyseal lesion with an associated soft tissue mass. Due to rapid growth, there is commonly “sunburst” periostitis or Codman’s triangles. X –rays are preferred for the initial diagnosis, but MR is superior to CT in the evaluation of the intra and extraosseous extent of disease. The metaphyseal location and age of the patient in question would be appropriate for osteosarcoma, but the smooth, benign reactive bone and lack of any destructive changes or a soft tissue mass are consistent with a non- aggressive process. C) Incorrect. Chondroblastoma is a rare, benign primary tumor of bone that most commonly seen in the second decade. It typically presents as a lytic lesion in the epiphysis or apophysis of a long bone with a well-defined sclerotic margin. Matrix calcification is present in up to 50%. MRI may show edema in the surrounding bone and soft tissues due to the prostaglandins secreted by the tumor and clinically they may mimic osteoid osteoma. Benign periostitis or joint effusions may be seen, most commonly when the lesion is located in the capital femoral epiphysis within the hip joint capsule. The lesion may appear expansile, most commonly when a secondary aneurysmal bone cyst is coexistent. Rarely chondroblastoma may metastasize to the lungs. Treatment is usually curettage with bone grafting with image-guided radiofrequency ablation being used at some centers. Although the age of the patient in the index case would be appropriate for chondroblastoma, the location in the Meta diaphysis of the femur and the large amount of reactive bone would not be. 2 American College of Radiology
  • 3. Section III – Musculoskeletal Radiology D) Correct. Osteoid osteoma is a benign bone-forming neoplasm consisting of a central core of vascularized osteoid surrounded by densely sclerotic bone. The clinical history is often suggestive, with pain, which is worse at night and relieved by prostaglandin inhibiting agents such as aspirin. Age at presentation is usually in the second or third decade. Lesions in long bones are commonly cortically based where they typically present as lucency, the “nidus”, which may or may not contain calcification. The nidus is usually located at the center of the reactive sclerotic bone. In the small bones of the hands and feet, the lesions tend to be intramedullary with an associated periosteal reaction. Subperiosteal lesions can be seen and may have less prominent reactive changes. The most common location is in the long tubular bones, typically in the diaphysis or metaphysis. Vertebral lesions, often associated with a painful scoliosis, are usually located in the posterior elements. MR imaging of osteoid osteoma shows edema in the bone and soft tissues, which may be deceptively aggressive in appearance. Scintigraphy has been used in the past for its high sensitivity but it remains low in specificity. Plain films usually show the benign reactive bone. High resolution CT is best for showing the nidus itself. CT may also be used for pre-operative localization or definitive treatment with radiofrequency ablation. Our case shows the classic appearance of osteoid osteoma with the dense but benign reactive bone with a subtle lucency on x-ray. The CT demonstrates a well-defined nidus with central calcification. This case was subsequently successfully treated with CT-guided radiofrequency ablation with complete resolution of symptoms. E) Incorrect. Ewing’s Sarcoma is a primary malignancy of bone that chiefly affects young children, often under the age of 10. It most commonly affects the femur and in general is more common in the lower part of the body. In long bones, the Meta diaphysis or diaphysis are typical locations in the bone. Radiographically the lesion is primarily lytic and may have a permeative appearance that may be mistaken for infection. A malignant periosteal reaction is present which may appear laminated (“onion-skin” pattern) and a large soft tissue mass is usually seen. The lesion appears central in the bone reflecting its origin from bone marrow. Ewing’s Sarcoma often presents with constitutional symptoms such as fever, which may delay diagnosis. Scintigraphy is sensitive but non-specific. Radiographs are the usual modality for primary diagnosis with MRI showing the extent of disease within the bone marrow and any associated soft tissue mass. MRI is also often used to monitor response to treatment with chemotherapy. CT is especially helpful when flat bones such as the pelvis or skull are involved. In our case, the lesion is cortically based with a very benign and dense periosteal reaction. This would not be consistent with a malignant process. The CT scan shows no soft tissue mass or disruption of soft tissue planes. Citations: Torriani M, Rosenthal DL. Percutaneous radiofrequency ablation of osteoid osteoma. Pediatric Radiology 32(8): 615-8, 2002. Ho AC, Horton KM, McCarthy EF, Fishman EK. The role of imaging in the diagnosis and management of osteoid osteoma: a pictorial review. Critical Reviews in Diagnostic Imaging 42(6): 357-77, 2001. Diagnostic In-Training Exam 2003 3
  • 4. Section III – Musculoskeletal Radiology Figure 2 91. You are shown an axial T2-weighted, fat suppressed MR image in a 25-year-old woman who presents with pain after kicking the ball while playing soccer (Figure 2). Which one of the following is the MOST likely diagnosis? A. Hamstring avulsion B. Insufficiency fracture C. Adductor strain D. Gluteal myositis E. Denervation injury 4 American College of Radiology
  • 5. Section III – Musculoskeletal Radiology Question #91 Findings: The image presented is a fat suppressed, T2 weighted axial MR image of the lower pelvis at the level of the ischial tuberosities. The image shows a focal area of increased T2 signal between the hamstring tendons on the right and the tip of the ischial tuberosity. The marrow signal in both inferior pubic rami is normal, as is the signal in the musculature. Rationales: A) Correct. Hamstring injuries are commonly seen in athletes such as hurdlers or those who participate in sports with powerful kicking. In children, these injuries are often associated with avulsions of the ischial apophysis; in young adults, tendon avulsions without underlying fractures may be seen. Radiographs are often normal in the absence of an associated fracture. MRI will show the relationship of the tendons to their attachment, associated muscle injuries, and the presence or absence of osseous pathology. In our case, the T2 weighted fat suppressed axial image shows high signal fluid between the hamstring tendons and the bone, with no such separation on the contralateral normal side. The inferior pubic rami are normal on both sides with no evidence for fracture. The muscles themselves are also normal. There is some fluid surrounding the right sciatic nerve, which is otherwise normal, explaining why this patient may present with complaints of sciatica due to irritation of the nerve related to its proximity to the tendon avulsion. These injuries are most often treated conservatively with the exception of apophyseal avulsions in childhood, which may require fixation. B) Incorrect. Insufficiency fractures are the result of normal stresses on bone that has lost its normal elastic resistance. The pelvis is a common location for these fractures, which are usually seen, in elderly, osteoporotic women. In particular, the inferior and superior pubic rami are often affected, with a subgroup of these patients having avulsion insufficiency fractures of the ischial tuberosity. In this case, the most common presentation is sciatica due to irritation of the nearby sciatic nerve. For this reason, diagnosis may be delayed as the potential for lumbar spine pathology is evaluated. These fractures are usually visible on radiography. In the very acute phase, non-displaced fractures may be difficult to identify. Scintigraphy is very sensitive but lacks specificity and anatomic resolution. MRI will show the edema and any associated tendon or muscle injuries. CT will best display the fracture, and is particularly helpful in excluding pathologic fractures in the sacrum. In our case, the inferior pubic rami are well seen and normal bilaterally, excluding the possibility of acute or subacute fracture. C) Incorrect. The adductor muscle group includes the adductor magnus, brevis and longus as well as the gracilis, pectineus and Sartorious muscles. These muscles principally take their origin from the pubic ramus and are located in the medial thigh, primarily acting to abduct the thigh although individual muscles in this group contribute to actions such as hip flexion and extension. Muscle injuries in general may be divided into contusions or strains, partial tears and complete tears or lacerations. Radiography is usually normal, but will show associated osseous injuries. MRI is the preferred imaging modality for evaluating muscle injuries. A muscle contusion or strain will appear as an intact muscle with increased T2 signal suggesting edema. Partial or complete tears can also be identified. In the acute phase, MRI may show hemorrhage at the site of injury. In the image shown for this question, some of the upper adductor musculature is visible, notably the adductor magnus and brevis. These muscles are entirely normal in their signal characteristics and morphology with no edema or loss of muscle bulk, effectively excluding a significant muscle injury. D) Incorrect. Myositis is a non-specific term indicating inflammation within a muscle. Etiologies include bacterial, viral, and parasitic infections, collagen vascular diseases such as SLE and even drug toxicities. Pyomyositis is a distinct entity, which is often related to staphylococcal infection. The imaging appearance of myositis is also non-specific. On MRI, muscles may have increased T2 signal and there may be loss of distinction between tissue planes on T1 weighted sequences. Contrast-enhanced studies, using CT or MRI, will show any associated abscesses such as those seen with pyogenic infections. Accurate diagnosis requires aspiration and culture of the recovered material. Depending on the location, ultrasound or CT can be used for imaged guided aspiration or drain placements. In our case, all of the muscles, including the gluteus group, are normal with no bright T2 signal except at the sight of the tendon avulsion, making myositis an extremely unlikely diagnosis in this example. Diagnostic In-Training Exam 2003 5
  • 6. Section III – Musculoskeletal Radiology E) Incorrect. Denervation injury to muscle can be the result of acute or chronic trauma to a nerve or other processes such as inflammatory neuropathies. Compressive neuropathies such as the anterior interosseous nerve syndrome or carpal tunnel syndrome also fall into this disease category. Initial imaging findings may be negative despite positive clinical examinations or studies such as EMG. As the disease progresses, the muscles that are innervated by the affected nerve may show some mild increased T2 signal. As the disease becomes chronic, there is often loss of muscle bulk and fatty replacement. In the case of large peripheral nerves such as the sciatic or median nerves, the abnormality in the nerve itself may be seen as increased size and T2 signal. In the case of smaller nerves, which cannot be easily resolved on imaging studies, the key to diagnosis is recognizing the pattern of muscle involvement relating to a specific nerve. In our case, the muscles are normal and even though there is some fluid surrounding the right sciatic nerve, the nerve itself is normal and symmetric with the opposite side. Citations: Brandser EA, el-Khoury GY, Kathol MH, Callaghan JJ, Tearse DS. Hamstring injuries: radiographic, conventional tomographic, CT, and MR imaging characteristics. Radiology 197(1): 257-62, 1995. 6 American College of Radiology
  • 7. Section III – Musculoskeletal Radiology Figure 3A Figure 3B 92. You are shown PA (Figure 3A) and oblique (Figure 3B) radiographs of the hand in a 40-year-old woman with hand pain. Which one of the following is the MOST likely diagnosis? A. Gout B. Systemic Lupus Erythematosus C. Erosive osteoarthritis D. Scleroderma E. Rheumatoid arthritis Diagnostic In-Training Exam 2003 7
  • 8. Section III – Musculoskeletal Radiology Question #92 Findings: PA and oblique radiographs of the hand show diffusely decreased bone density in this 40-year-old woman. On the oblique view, there are multiple subluxed MCP and IP joints with dislocation of the 5th PIP joint on the oblique image with near complete reduction on the PA view. No erosions or productive changes are seen. Rationales: A) Incorrect. Gout is a crystalline deposition disease. Both primary and secondary gout are related to hyperuricemia, with the primary form representing an inborn error of metabolism and secondary gout arising from altered uric acid metabolism associated with other clinical disorders. Primary or idiopathic gout is much more common in men and typically presents in the fifth decade. It often begins as a monoarticular or oligoarticular disorder, progressing to involve more joints over time. The most commonly involved joint is the first metatarsophalangeal joint, which is, altered 75-100% of patients with gout. Most patients with biochemical and clinical evidence of gout will not have bone changes due to early treatment of the metabolic disorder. In patients who do manifest radiographic changes, the findings are usually classic and diagnostic of gout. The joint space is well preserved, with erosions in intra-articular and para-articular locations. These erosions are well defined with sclerotic margins and overhanging edges in up to 40%. The overhanging edges may be associated with gouty tophi in the adjacent soft tissues, which often show increased density or even calcification on x-rays. Bone density is usually preserved. The distribution in the skeleton is typical, with the feet most commonly involved. Hand and wrist, the knee and elbow are also usual sites. Cross sectional imaging is rarely contributory in the evaluation of gout. MRI may be helpful in early gout to identify synovitis and early erosions as well as the extent of soft tissue involvement. Gout would not be an appropriate consideration in the case in question. There are no erosions or other destructive changes present, effectively excluding a radiographic diagnosis of gout. B) Correct. Systemic lupus erythematosus is an autoimmune connective tissue disorder affecting multiple organ systems. In the musculoskeletal system, common manifestations are a deforming, symmetric polyarthritis, myositis, tendon weakening, rupture, and osteonecrosis. As with most collagen vascular diseases, adult women are most commonly affected. The hallmark of the arthritis associated with SLE is deformity without destruction. The small joints of the hand are characteristically involved with multiple subluxed or even dislocated joints, which are easily and usually completely reducible. In fact, the positioning of the patient for the PA radiograph of the hand may itself reduce the subluxations making the disease less prominent. A relaxed oblique or “ball- catcher’s” view often shows the subluxations to better advantage as in our case. Periarticular osteopenia is common and reminiscent of rheumatoid arthritis, but the complete lack of any erosive changes should help to distinguish these entities. Osteonecrosis may be present in the form of avascular necrosis, bone infarcts, or both but this usually involves long bones rather than the small bones of the hands. Jacoud’s arthropathy, a sequela of rheumatic fever, has an appearance that is identical radiographically to SLE. Fortunately, this has become very rare in the era of antibiotic treatment of streptococcal infections, limiting differential diagnostic considerations. C) Incorrect. Erosive osteoarthritis is an inflammatory variant of osteoarthritis, which is characterized by a combination of erosive and productive changes typically in the DIP joints of the hands. This disease usually affects middle-aged women. While other changes of degenerative osteoarthritis may be present within the other joints of the hand and wrist, the DIP involvement is usually strikingly worse. The primary differential considerations are other types of inflammatory arthritis, including psoriatic arthritis, Reiter’s syndrome, and even metabolic disorders such as hyperparathyroidism. What distinguish erosive osteoarthritis from these entities are the distribution of the radiographic changes and the lack of associated systemic disease. Psoriatic arthritis may affect women in the same age group. This is usually not confined to the DIP joints and the pattern of erosive changes is different. Reiter’s syndrome usually affects younger men and more typically involves the lower extremities. Rheumatoid arthritis spares the DIP joints in nearly all cases. Other forms of inflammatory and erosive arthritis also have typical radiographic patterns of disease. In our case, the complete lack of any erosive or productive changes in the interphalangeal joints excludes erosive osteoarthritis as a possibility. 8 American College of Radiology
  • 9. Section III – Musculoskeletal Radiology D) Incorrect. Scleroderma, or progressive systemic sclerosis, is a rare disorder of connective tissue affecting multiple organ systems most commonly affecting women in the third to fifth decades. In the musculoskeletal system, the hands are the most common sites of involvement, with changes most pronounced in the digits. Progressive atrophy of the soft tissues at the tips of the fingers creates a characteristically conical appearance to the finger. Progressive erosion and resorption of the ungual tuft of the fingers is also commonly seen with amorphous calcifications seen in the soft tissues. With time, the more proximal bones of the fingers may be involved as well with further resorption giving a “pencil” appearance to the digit. The feet are usually not affected to the same degree as the hands. Other sites where bony changes can be seen include the ribs, spine, and mandible. Soft tissue calcifications often are more diffuse and periarticular tumoral calcinosis may be seen. In the index case, the bones of fingers are normal except for their alignment and no soft tissue resorption or calcifications are seen. E) Incorrect. Rheumatoid arthritis is a relatively common inflammatory arthritis affecting synovial joints, bursae, and tendon sheaths. The primary process is one of synovial inflammation with secondary affects on the underlying bone and cartilage. The small joints of the hands are commonly involved as are the cervical spine, feet and other sites. In the early stages of disease the primary finding may be soft tissue swelling over the MCP joints and ulnar styloid representing synovitis. Periarticular osteopenia may also be present early on reflecting hyperemia at the inflamed joints. With progression, erosions are seen in characteristic locations such as the ulnar styloid and MCP joints but no productive changes are present. The DIP joints are uniformly spared. As the inflamed synovium destroys ligaments and tendons, subluxations such as the swan’s neck deformity become common. Although the subluxations in our case could be seen in rheumatoid arthritis, the complete lack of any destructive changes or soft tissue swelling makes this much less likely. Citations: Brower A, Flemming DJ, Bralow L. Arthritis in Black and White. 2cd edition, W.B. Saunders, Philadelphia, 1997. Diagnostic In-Training Exam 2003 9
  • 10. Section III – Musculoskeletal Radiology Figure 4A Figure 4B 93. A 45-year-old man presents with ankle pain after playing basketball. You are shown sagittal T1 (Figure 4A) and T2 and sagittal-T2 (Figure 4B) weighted MR images of the ankle. Which one of the following is the most likely diagnosis? A. Pilon fracture B. Achilles tenosynovitis C. Achilles tendon rupture D. Os trigonum syndrome E. Calcaneal stress fracture 10 American College of Radiology
  • 11. Section III – Musculoskeletal Radiology Question #93 Findings: Sagittal T1 and fat suppressed T2 weighted MR images of the ankle show disruption of the Achilles tendon with retraction of several centimeters. The visualized distal tendon is thickened with increased intra substance signal. There is increased T2 signal in the gap and the adjacent soft tissues. A small ankle effusion is present. Osseous structures are normal. Rationales: A) Incorrect. Pilon fractures of the ankle result from pronation-dorsiflexion injuries, which drive the talar dome into the tibial plafond. Included in this complex are an oblique fracture of the medial malleolus and an intraarticular fracture of the distal tibia, which may have more than one part. A fibular fracture may also be present. These are uncommon fractures, representing less than 0.5% of all ankle fractures. Treatment decisions are based on the degree of comminution of the fracture and the extent of intraarticular involvement and displacement. Radiography and knowledge of the mechanism of injury are usually diagnostic, but CT scanning with multiplanar reformatting may be helpful to assess f small intra-articular fragments and to better quantify the degree of displacement and articular incongruence. MRI is generally not contributory to management in the acute phase. In our case, the MRI of the ankle shows a normal tibia and talar dome, excluding fracture from the diagnosis. B) Incorrect. The Achilles tendon is the largest tendon in the body. It represents the confluence of the gastrocnemius and soleus tendons, inserting on the posterior calcaneus. In adults, the Achilles tendon is approximately 10-15cm long. Unique among the tendons of the ankle, the Achilles tendon does not have a synovial-lined tendon sheath, so that tenosynovitis involving the Achilles tendon is not a possibility. Rather, it is covered by a peritenon. Peri- or para tendonitis may be seen within the surrounding soft tissues, often associated with tendinoplasty or partial tears of the tendon itself. Retrocalcaneal bursitis may also be seen just anterior to the insertion of the Achilles tendon on the calcaneus. Our case shows the clear discontinuity of the Achilles tendon with surrounding fluid related to the injury, but is not suggestion of a separate inflammatory process. C) Correct. Achilles tendon rupture is most common in men between the ages of 30 and 50. The typical scenario is a “weekend warrior” who participates in a sport such as basketball, which uses sudden, forceful dorsiflexion or push off of the foot. Clinically there is sudden onset of pain and soft tissue swelling with an inability to stand on tiptoe on the affected side. Radiographs are usually obtained to exclude fracture and may show loss of the soft tissue planes surrounding the Achilles tendon or may even suggest disruption and retraction of the tendon itself. MRI definitively shows the disruption as well as its location and the degree of retraction. The most common site of a complete tear is approximately 2-6 cm proximal to the insertion of the tendon on the calcaneus. This site is vulnerable both to partial and complete tears as a relatively avascular portion of the tendon. In addition to trauma in unconditioned individuals, Achilles tendon rupture may be associated with chronic tendinoplasty and partial tears of the tendon, rheumatoid arthritis, SLE, and the use of local or systemic corticosteroids. MRI may show hemorrhage or fluid in the acute phase as well as the discontinuity of the tendon. When the problem is subacute or chronic, as in our case, the MRI shows discontinuity and some mild retraction as well as a thickened Achilles tendon with increased signal in the distal portion due to underlying tendinoplasty. Increased T2 signal is seen surrounding the ruptured tendon consistent with edema and fluid. D) Incorrect. The Os Trigonum or talar compression syndrome is a pain syndrome involving the posterior ankle. The os trigonum is an accessory ossicle just posterior to the talus at the ankle. When fused to the posterior talus it is referred to as Stieda’s process. The flexor hallucis longus (FHL) tendon lies immediately adjacent to the os trigonum. In some cases where the os trigonum is enlarged or is relatively more mobile, irritation of the FHL tendon or the posterior talus itself may be seen, with tenosynovitis of the FHL or even partial tears of the tendon resulting. Radiography will show the enlarged ossicle, which may appear irregular, but MRI is diagnostic, showing the edema surrounding the os trigonum and posterior talus and the associated abnormalities in the FHL tendon. In our case, the posterior talus is normal with no edema and the FHL tendon is not included on the images shown. Diagnostic In-Training Exam 2003 11
  • 12. Section III – Musculoskeletal Radiology E) Incorrect. Stress fractures are the result of repetitive loading on bone, which may be normal or abnormal. Fatigue fractures involving normal bone may be due to a novel, strenuous activity, which places repetitive stress on a specific bone; an example would be the march fractures of the metatarsal seen in new military recruits. In the case of abnormal underlying bone, such as osteoporosis, the term insufficiency fracture may be applied. Common sites for insufficiency fractures include the pelvis and calcaneus. In the calcaneus, radiographs show the underlying osteopenia with crescentic area of sclerosis usually in the posterior calcaneus. The sclerosis probably represents a combination of impaction and healing. In the very acute phase, these fractures may be radiographically occult. In this case, MRI would show the reactive edema on T2 weighted imaging as well as the fracture itself, which appears as a linear area of low signal intensity on T1 weighted images. Scintigraphy and MRI have similar sensitivity, but MR is significantly more specific. CT with multiplanar reformatting will also show the fracture before it is radiographically evident. In our case, the calcaneus is normal on both T1 and T2 weighted images with normal marrow signal throughout and no edema to suggest fracture. Citations: Schepsis AA, Jones H, Haas AL. Achilles tendon disorders in athletes. American Journal of Sports Medicine 30(2): 287-305, 2002. Movin T, Kristoffersen-Wiberg M, Rolf C, Aspelin P. MR Imaging in chronic Achilles tendon disorder. Acta Radiologica 39(2): 126-32, 1998. 12 American College of Radiology
  • 13. Section III – Musculoskeletal Radiology Figure 5A Figure 5B Figure 5C 94. This 50-year-old man presented with a mass along his right chest wall. You are shown coronal T1 (Figure 5A), and fat-saturated post gadolinium T1 (Figure 5B) weighted images and a non-contrast axial CT scan (Figure 5C) obtained 4 weeks after the MRI. Which one of the following is the MOST likely diagnosis? A. Myositis ossificans B. Malignant fibrous histiocytoma C. Desmoid tumor D. Chondrosarcoma E. Intramuscular myxoma Diagnostic In-Training Exam 2003 13
  • 14. Section III – Musculoskeletal Radiology Question #93 Findings: Coronal T1 and fat suppressed T1 post-gadolinium MR images of the right chest wall show a mass within the lateral chest wall, which is isointense to muscle on the non-contrast examination and shows avid enhancement with intravenous gadolinium. A small central area in the mass does not enhance and may be necrotic or cystic. A single axial non-contrast CT image obtained four weeks later shows benign, peripheral calcification within the teres major muscle. There has been no interval change in the size of the mass. Rationales: A) Correct. Myositis Ossificans (MO) is a localized soft tissue calcification most often related to antecedent injury. Up to 75% of patients who develop MO can relate a clear history of trauma to the affected area. Other causes may include burns, neurologic conditions, and systemic disorders associated with soft tissue calcification. Rarely the lesion may develop spontaneously. Common sites of MO include areas exposed to injury such as the buttocks, elbow, thigh, and calf. A soft tissue mass appears at the site of injury within about 10 days, with calcification usually appearing by 6 weeks. The calcification typically forms at the periphery of the soft tissue lesion, forming a smooth, complete border when mature. The peripheral calcification is an important feature distinguishing developing MO from malignancies such as osteosarcoma or chondrosarcoma. If the lesion is imaged with MR early in the process, the appearance can be very worrisome with avid contrast enhancement and features consistent with a soft tissue sarcoma such as a malignant fibrous histiocytoma. Even biopsy may be misleading if obtained from the cellular central portions of the mass. As with imaging, the more mature peripheral zone shows the benign nature of the lesion. Radiography will often show the mature peripheral calcification well enough for diagnostic confidence. If there is any question regarding the nature of the calcification (with an appropriate clinical history, of course) CT can be obtained which will show the benign calcification to best advantage. In our case, the patient was involved in a car accident with an injury to his chest wall approximately two weeks prior to the MRI. The MRI was felt to be worrisome for malignancy, but because of the history of trauma, biopsy was not immediately performed. CT obtained 4 weeks later shows the mature calcification of MO and no further evaluation was required. B) Incorrect. Malignant fibrous histiocytoma (MFH) is the most common soft tissue sarcoma in adults. Rarely (5%) it may arise as a primary bone tumor. MFH presents as a painless soft tissue mass. Radiographs may show the tumor in the soft tissue if large, but the preferred modality for evaluation of soft tissue tumors is MRI. On MRI, the mass will be low in signal intensity on T1 weighted imaging and bright on T2. If contrast is administered, there is usually avid and homogeneous enhancement. In very large tumors there may be non- enhancing areas centrally representing necrosis. MRI best demonstrates the anatomic compartments involved and the relationship of the tumor to neurovascular structures, bones and joints. CT with contrast may be used if the patient is unable to undergo MRI. Ultrasound may be helpful for image-guided biopsy. Imaging is non- specific; high-grade liposarcomas and other types of primary sarcomas and metastases may have similar imaging features. MFH may rarely calcify, with the calcification located within the mass and appearing irregular. If the mass is adjacent to a bone, that bone may undergo pressure erosion. MFH was a consideration when the MRI was obtained in our patient, but because of the history of trauma, biopsy was deferred in favor of a follow-up CT. C) Incorrect. Desmoid tumor is the name given to a benign fibrous proliferation arising in the abdominal and extra-abdominal musculature, often growing slowly and engulfing the surrounding tissue in an insidious fashion. While histologically benign, these tumors commonly recur after excision and are difficult to treat. As with most of the fibromatoses, they are locally aggressive and may cause erosion of adjacent bones with a periosteal reaction sometimes seen. They are usually solitary with a predilection for the shoulder girdle. Radiography may show the soft tissue mass if large enough as well as any effects on adjacent bone, but MRI is the preferred modality for diagnosis, with the dense fibrous tissue having characteristic signal intensity which is low on both T1 and T2 weighted imaging. If intravenous gadolinium is given, there is usually marked enhancement throughout the lesion. Calcifications may be seen but are rare. In our case, the signal intensity of the lesion on the unenhanced T1 image is not as low as would be expected with a desmoid tumor. The post-gadolinium image shows the mass 14 American College of Radiology
  • 15. Section III – Musculoskeletal Radiology to be well defined; desmoid tumors often have a less well-defined, more infiltrative appearance. The CT showing peripheral calcification in the mass is diagnostic of myositis ossificans and would be extremely unusual for a desmoid. The combination of history and the CT appearance effectively excludes the diagnosis of a desmoid tumor. D) Incorrect. Chondrosarcomas are tumors of cartilaginous origin, which may occur primarily, or secondary to pre-existing lesions such as an enchondroma or osteochondroma. They may also be categorized according to their location in bone or by their histologic characteristics. Extraskeletal chondrosarcomas are rare. The most common location for chondrosarcoma is in the long tubular bones, with the femur being the most frequently affected bone. They are usually located in the metaphysis but extension to the epiphysis can be seen. Most chondrosarcomas are of low histologic grade. Radiographically, these tumors may show a primarily lytic area with endosteal scalloping and chondroid calcification within the lesion. Cortical thickening may also be seen. Higher grade or dedifferentiated chondrosarcomas will have a more aggressive appearance, with cortical breakthrough and large soft tissue masses. Radiography is the primary diagnostic modality. When the flat bones such as the pelvis and scapula are involved, CT may be preferred both to define the extent of the lesion and to characterize the calcifications. MRI may also be used to evaluate the extent of soft tissue and marrow involvement and to establish which anatomic compartments are involved for surgical planning. In the unusual case of an extraskeletal chondrosarcoma, a soft tissue mass with c chondroid calcification is seen. Our case shows smooth peripheral ossification, making a cartilaginous process extremely unlikely. E) Incorrect. Intramuscular myxoma is an uncommon benign soft tissue mass. As its name would suggest, the mass is located within a muscle, most often in an extremity. Radiography rarely shows these lesions. MRI will show a well-circumscribed mass within a muscle, which is very low in signal intensity on T1, and very bright on T2 weighted images. Myxomas rarely enhance except for a smooth, peripheral rim. These lesions almost never calcify. In our case, although the mass is located within the chest wall musculature, the mass enhances homogeneously which would be inconsistent with a myxoma, as would be the peripheral calcification. Citations: Jepsen MC, Graham SM. Traumatic myositis ossificans of the levator scapulae muscle, American Journal of Otolaryngology 19(5): 345-348, 1998. Resnick, D. “Soft tissues” in Resnick, D. Bone and Joint Imaging, 2cd edition, 1261-1263, W.B. Saunders, Philadelphia, 1996. Diagnostic In-Training Exam 2003 15